LSAT and Law School Admissions Forum

Get expert LSAT preparation and law school admissions advice from PowerScore Test Preparation.

 pacer
  • Posts: 57
  • Joined: Oct 20, 2014
|
#17585
I am confused with this question. I don't see why answer choice C is incorrect.

This is a conditional stimulus and I diagrammed it as follows

Supports new tax plan -> no chance of being elected

truly understands economics -> doesn't support new tax plan

chance of being elected -> truly understands economics

In summary

chance of being elected -> truly understands -> doesn't support new tax plan

When going through the answer choices,

A - restates something that is already stated in the stimulus
B - restates something that is already stated in the stimulus

C - since doesn't support new tax plan is a necessary condition, it follows that a sufficient condition (chance of being elected) may or may not happen

I though that this is a flaw

for choice D, the same reasoning follows

since doesn't support new tax plan is a necessary condition, it follows that a sufficient condition (truly understands economics) may or may not happen

Not sure about choice E

Can you please go through this problem step by step? I don't know where I am going wrong.

Thanks
 Emily Haney-Caron
PowerScore Staff
  • PowerScore Staff
  • Posts: 577
  • Joined: Jan 12, 2012
|
#17591
Hi pacer,

Here's my diagram:
support new tax plan :arrow: NOT elected
truly understands economics :arrow: NOT support new tax plan
(contrapositive: support new tax plan :arrow: NOT truly understands economics)

CONCLUSION:
elected :arrow: truly understands economics

The two pieces of the stimulus, above, cannot be connected to each other; supporting the new tax plan means BOTH that someone cannot be elected and does not truly understand economics, but that doesn't tell us anything about people who DO have a chance of being elected.

You're right that A just reiterates the stimulus. B is just the opposite of the conclusion, but doesn't explain anything. C could be true, but is irrelevant to the conclusion, which is talking about people who COULD be elected. E is accurate reasoning, but again is irrelevant to the conclusion.

That leaves us with D. The flaw in the conclusion is that it assumes that just because you do not support the new tax plan, and therefore have a chance of being elected, means you also MUST understand economics. That is a mistaken reversal of the second premise I diagrammed at the top. It is possible that someone who doesn't support the new tax plan also does not truly understand economics, but could still have a chance at being elected.

Does that help?
 pacer
  • Posts: 57
  • Joined: Oct 20, 2014
|
#17603
Can you give some tips on how to determine when pieces of the stimulus fit together and when they do not?

This question, I still have trouble seeing why and how the stimulus don't fit together.
 pacer
  • Posts: 57
  • Joined: Oct 20, 2014
|
#17605
From my understanding, given the premises, we know about people who do not have a chance of being elected and the conclusion is being made about people who have a chance of being elected. So, I get that choice C is talking about people who have no chance of being elected. But then choice D doesn't talk about people's chance of being elected at all. Isn't D just stating the opposite of premise #2?

premise 2: understands -> doesn't support

choice D: doesn't support -> doesn't understand
 Nikki Siclunov
PowerScore Staff
  • PowerScore Staff
  • Posts: 1362
  • Joined: Aug 02, 2011
|
#17619
Hi pacer,

Your first question was a bit vague, but I think I understand the issue: you're trying to match keywords. That will sometimes help on the test, but - more often than not - it will not. The key is to understand precisely what's the problem with this line of reasoning. Knowing that the author makes a Mistaken Reversal is not enough. You need to also realize why the Mistaken Reversal is an unwarranted inference, i.e. what possibility does it fail to take into account.

The premises in the argument, when re-worded, can be diagrammed as follows:

Elected :arrow: NOT support tax plan
Truly UE :arrow: NOT support tax plan

These premises do not produce a chain relationship; the author, however, thought that they do:

Elected :arrow: NOT support tax plan :arrow: Truly UE

Essentially, the author made a Mistaken Reversal of the second premise:

Stimulus: Truly UE :arrow: NOT support tax plan

Mistaken Reversal: NOT support tax plan :arrow: Truly UE

What possibility does the MR ignore? That some people who do not support the tax plan may not truly understand economics. This prephrase is consistent with answer choice (D). Essentially, the Mistaken Reversal ignores the possibility that we can satisfy a necessary condition (not support tax plan) without satisfying the sufficient condition (truly understand economics).

Let me give you another example. Let's say that the LSAT is required for admission into law school (premise), and conclude that anyone who takes the LSAT is guaranteed admission:

Premise: Admission :arrow: LSAT

Conclusion: LSAT :arrow: Admission

My conclusion is clearly a Mistaken Reversal of my premise. What possibility does it ignore? That some people who take the LSAT may still not be admitted.

Hope this clears things up!
 lsatopocalypse
  • Posts: 12
  • Joined: Nov 17, 2014
|
#20162
hello, I understand this is an old test but i believe it is still quality practice. I am extremely confused by the entire question and would really appreciate if someone could do a conditional breakdown of the question. I struggle with conditional statements.
 Ladan Soleimani
PowerScore Staff
  • PowerScore Staff
  • Posts: 43
  • Joined: Oct 08, 2015
|
#20163
Hello Lsatapocalypse

Old tests are definitely quality practice! This is a tricky question, especially if you are not completely comfortable with conditional reasoning. The wording of the question stem also adds to the complexity of this question.
Here is the breakdown:

Premise 1: Anyone who supports the new tax plan has no chance of being elected.
..... STP :arrow: E

Premise 2: Anyone who truly understands economics would not support the tax plan.
..... UE :arrow: STP

Conclusion: Only someone who truly understands economics would have any chance of being elected.

..... E :arrow: UE

So altogether:

..... STP :arrow: E ..... ..... contrapositive: ..... E :arrow: STP
..... UE :arrow: STP ..... contrapositive: ..... STP :arrow: UE
..... _______________
..... E :arrow: UE

The author is trying to chain the two premises together to reach the conclusion, but they are actually unable to be connected. In order to properly connect being elected from the first premise to understanding economics from the second premise, the author would need a premise saying:

..... STP :arrow: UE

which is a Mistaken Reversal of the second premise.

According to the question stem you are looking for a possibility that the author ignores, which essentially means the author is making an assumption. In this case the assumption that the author is making is the Mistaken Negation above: Anyone who does not support the tax plan understands economics. If this were true, the argument would be valid, but we have no reason to believe it is true. With this assumption, the author is overlooking the opposite possibility, that some people who do not support the tax plan do not understand economics:

..... Some STP :arrow: UE

which matches with answer choice (D)

I hope this helps!
Ladan
 lsatopocalypse
  • Posts: 12
  • Joined: Nov 17, 2014
|
#20204
Ladan, Thanks for taking the time to address my question. I am still not quite understanding it. I understand that the flaw is a mistaken reversal of the second premise, but to correctly solve this problem (and others like it) would one want to always default to finding the opposite of the mistaken reversal, in this case: does not support tax plan ----> does not understand econ?

Would reviewing the formal logic section of the logical reasoning bible help me to better understand the logic of this particular question type?
 Nikki Siclunov
PowerScore Staff
  • PowerScore Staff
  • Posts: 1362
  • Joined: Aug 02, 2011
|
#20208
Hi Lsatapocalypse,

Let me jump in real quick and offer some advice. Yes, this is a challenging question, in part because you first need to understand what the logical flaw is, and also because you need to be comfortable with the many different ways in which that flaw can be described. Let me offer an analogous example that may be easier to understand:
If you want to go to law school, you need to go to college first. Law school, however, is not a prerequisite to becoming a doctor. Therefore, you need not go to college to become a doctor.
When diagrammed, the argument would look like this:
Premise 1: Law :arrow: College
Premise 2: Doctor :arrow: Law
Conclusion: Doctor :arrow: College
This conclusion is flawed for the same exact reason the conclusion in the stimulus you asked about is flawed: both authors assume that the two premises can be connected in a way that logically validates the conclusion:

Doctor :arrow: Law :arrow: College

In creating this chain, the author Mistakenly Negated the first premise, believing - incorrectly - that if you don't want to go to law school, you don't need to finish college (unwarranted assumption: Law :arrow: College). In other words, the author assumes that only those who want to go to law school should finish college (College :arrow: Law). Remember that the Mistaken Negation and the Mistaken Reversals are contrapositives of each other. Both errors are essentially the same.

There are many ways in which test makers can describe this flaw. Let me give you a few examples:

The author assumes, without justification, that you should not finish college unless you want to go to law school.
The author presumes, without justification, that only those who want to go to law school should finish college.

The author confuses a necessary condition (i.e. college) for a sufficient one.

The author confuses a sufficient condition (i.e. law school) for a necessary one.

The author ignores the possibility that finishing college may be a prerequisite for career paths other than law school.

The author overlooks the possibility that some people who don't want to go to law school may still need to finish college.

The author assumes that going to med school/business school/etc. does not require finishing college
As you can see, there are many ways of expressing the same type of logical flaw; however, if you understand conditional reasoning well enough to identify the error in the original argument, you should be comfortable with a variety of ways of expressing it. Conditional Reasoning is covered extensively in Lesson 2 of the Full Length course, and also in the Logical Reasoning Bible.

Hope this helps a bit!
User avatar
 Dave Killoran
PowerScore Staff
  • PowerScore Staff
  • Posts: 5853
  • Joined: Mar 25, 2011
|
#20809
Hi Ryan,

It looks like you understood the argument, but ran into problems with the nature of the flaw. As a review, the first sentence in this argument contains two premises and a conclusion, in that order:

  • Premise: SNTP :arrow: CE

    Premise: UE :arrow: SNTP


    Conclusion: CE :arrow: UE

The two premises can be combined, after being manipulated via a contrapositive:

  • ..... ..... CE :arrow:

    Premise: ..... ..... ..... SNTP

    ..... ..... UE :arrow:


    Conclusion: CE :arrow: UE
In that premise diagram, both arrows terminate at SNTP.

When you look at the above, you can see there is a problem. The conclusion links CE and UE in an arrow relationship, but the premises don't support that relationship. So, the first question is, what would we have to add to the premises to get the conclusion? Well, we know the conclusion starts with CE and ends with UE. And we have CE :arrow: SNTP in the contrapositive of the first premise. So, if we had an arrow from SNTP to UE, we'd be good because that would create CE :arrow: SNTP :arrow: UE, which would result in the conclusion CE :arrow: UE.

Well, that relationship (SNTP :arrow: UE) is exactly what the author thought was occurring, because why else would he or she draw that conclusion? So, now we know what the author was thinking (and we know it was erroneous), and conveniently the question stem asks for the flaw, specifically what was it that the author ignored? The author thought that all of those who do not support the new tax plan truly (SNTP) understand economics (UE), but it is in fact possible that not all people who do not support the tax plan truly understand economics (this is because that arrow from SNTP to UE doesn't really exist—the author just assumed it does). And, when reworded ("not all" into "some are not"), that's exactly what answer choice (D) says.

It's a tricky one, but the kind of question that is really useful to study since it could easily reappear, and the steps required to the manipulate the argument show you how the test makers think. So, take some time with that one and really manipulate it around (as an exercise, maybe take the CP of the second premise instead of the first and see if you can do the reasoning that way).

Please let me know if that helps. Thanks!

Get the most out of your LSAT Prep Plus subscription.

Analyze and track your performance with our Testing and Analytics Package.